FMJ - SP - 2012 - R1 - 1

Questão 29

Homem de 55 anos, portador de câncer de pulmão, apresenta-se no serviço de emergência com queixa de tosse produtiva e fraqueza. Há 1 dia, apresentou 2 episódios de temperatura de 38,8ºC. Realizou há 10 dias seu quarto ciclo de quimioterapia. Negou quimioprofilaxia com quinolona. Ao exame físico: afebril, regular estado geral, FC = 82 bat/min; PA = 110 x 70 mmHg; pele, cavidade oral, pulmão, períneo e sítio de inserção de cateter sem alterações. Laboratório: Rx de tórax normal; função renal e hepática normais e urina I dentro da normalidade. Hemograma com Hb = 9,6 gm/dl; Ht = 27%; leucócitos totais = 200 mm³ e neutrófilos = 140 mm³. Plaquetas de 50.000. Qual é a melhor conduta a ser seguida?
Anterior Próxima

Comentários